- PowerScore Staff
- Posts: 5972
- Joined: Mar 25, 2011
- Sat Jan 21, 2012 12:00 am
#41159
Complete Question Explanation
(The complete setup for this game can be found here: lsat/viewtopic.php?t=11758)
The correct answer choice is (C)
As in previous questions, J and K play a key role in this question. If we need to place two students in such a way that the composition of every class is determined, we will need to address which of J and K will be in the level 1 class. Thus, the correct answer is likely to include one of J and K. Answer choices (B) and (E), neither of which include J or K, are therefore less likely to be correct.
Of the three remaining answers, we can apply a different analysis, but one that is equally effective. J and K are somewhat similar, and the three students in answer choice (A), (C), and (D) besides J and K are F, H, and L. In examining F, H, and L, note that F and L are virtually identical in their basic characteristics: both are at the end of the same chain, behind K. Thus, pairing K with F or L (as do (A) and (D)) is likely to produce a similar result. The Uniqueness Theory of Answer Choices suggests that two answers with similar characteristics are unlikely to be correct. On the other hand, H sits in the middle of a different chain and automatically impacts N, who has a lower score. Thus, H appears to be more unique than F and L, and for our purposes this suggests that answer choice (C) is most likely to be correct.
Let’s take a closer look at answer choice (C), then. If H and J are in the level 2 class, then M must be in the level 2 class with them. Consequently, K must be in the level 1 class, and N, F, and L must be in the level 3 class. Thus, answer choice (C) is correct.
(The complete setup for this game can be found here: lsat/viewtopic.php?t=11758)
The correct answer choice is (C)
As in previous questions, J and K play a key role in this question. If we need to place two students in such a way that the composition of every class is determined, we will need to address which of J and K will be in the level 1 class. Thus, the correct answer is likely to include one of J and K. Answer choices (B) and (E), neither of which include J or K, are therefore less likely to be correct.
Of the three remaining answers, we can apply a different analysis, but one that is equally effective. J and K are somewhat similar, and the three students in answer choice (A), (C), and (D) besides J and K are F, H, and L. In examining F, H, and L, note that F and L are virtually identical in their basic characteristics: both are at the end of the same chain, behind K. Thus, pairing K with F or L (as do (A) and (D)) is likely to produce a similar result. The Uniqueness Theory of Answer Choices suggests that two answers with similar characteristics are unlikely to be correct. On the other hand, H sits in the middle of a different chain and automatically impacts N, who has a lower score. Thus, H appears to be more unique than F and L, and for our purposes this suggests that answer choice (C) is most likely to be correct.
Let’s take a closer look at answer choice (C), then. If H and J are in the level 2 class, then M must be in the level 2 class with them. Consequently, K must be in the level 1 class, and N, F, and L must be in the level 3 class. Thus, answer choice (C) is correct.
Dave Killoran
PowerScore Test Preparation
Follow me on X/Twitter at http://twitter.com/DaveKilloran
My LSAT Articles: http://blog.powerscore.com/lsat/author/dave-killoran
PowerScore Podcast: http://www.powerscore.com/lsat/podcast/
PowerScore Test Preparation
Follow me on X/Twitter at http://twitter.com/DaveKilloran
My LSAT Articles: http://blog.powerscore.com/lsat/author/dave-killoran
PowerScore Podcast: http://www.powerscore.com/lsat/podcast/